Suppose that an insurance company has 190 policy holders, and pays out $38000 in claims in a given year. The company also spends $1750 for marketing, $6000 for labor, and wants to earn a 20% profit. Given we do not have risk groups, what would a fair price be for annual premiums for the policy holders to pay?

Answers

Answer 1

The fair price for annual premiums for the policy holders to pay is $288.42

To determine the fair price for annual premiums, we need to take into account the company's expenses and desired profit. The company's total expenses can be calculated as the sum of the claims paid, marketing expenses, and labor expenses:

Total expenses = Claims paid + Marketing expenses + Labor expenses

Total expenses = $38000 + $1750 + $6000

Total expenses = $45750

To calculate the fair price for annual premiums, we need to add the desired profit to the total expenses and divide by the number of policy holders:

Fair price = (Total expenses + Desired profit) / Number of policy holders

Fair price = ($45750 + 20% of $45750) / 190

Fair price = ($45750 + $9150) / 190

Fair price = $54900 / 190

Fair price = $288.42

To know more about annual premiums, refer here:

https://brainly.com/question/9834518#

#SPJ11


Related Questions

The answer and what is the value of a

Answers

Answer:a=40

Step-by-step explanation:

angles on a straight line add to 180. This means that the missing angle that isn't a is 40. Angles in a triangle add to 180 so a=40

In a recent Game Show Network survey, 30% of 5000 viewers are under 30. What is the margin of error at the 99% confidence interval? Using statistical terminology and a complete sentence, what does this mean? (Use z*=2. 576)



Margin of error:



Interpretation:

Answers

The margin of error at the 99% confidence interval is 0.018 or 1.8%.

Interpretation: This means that if we were to repeat the survey many times, about 99% of the intervals calculated from the samples would contain the true proportion of viewers under 30 in the population, and the margin of error for each interval would be no more than 1.8%.

The margin of error is the amount by which the sample statistic (in this case, the proportion of viewers under 30) may differ from the true population parameter.

Using the given formula for margin of error:

Margin of error = z* * sqrt(p*(1-p)/n)

Where:

- z* is the z-score corresponding to the confidence level (99% in this case), which is 2.576

- p is the proportion of viewers under 30, which is 0.3

- n is the sample size, which is 5000

Substituting these values, we get:

Margin of error = 2.576 * sqrt(0.3*(1-0.3)/5000) = 0.018

To know more about margin of error refer to

https://brainly.com/question/10218601

#SPJ11

Prepare the operating activities section of the statement of cash flows for peach computer using the indirect method. (list cash outflows and any decrease in cash as negative amounts.)

Answers

Cash outflows and decreases in cash are reported as negative amounts in the operating activities section of the statement of cash flows for Peach Computer using the indirect method.

How to prepare the operating activities section of the statement of cash flows for Peach Computer using the indirect method?

The operating activities section of the statement of cash flows for Peach Computer using the indirect method would include the following cash inflows and outflows:

Cash inflows:

Sales revenue from the sale of computersCash received from customers for computer repairs and servicesInterest received on loans or investments

Cash outflows:

Payments to suppliers for inventory purchasesPayments to employees for salaries and wagesPayments for operating expenses such as rent, utilities, and advertisingPayments of income taxesPayments of interest on loansPayments to creditors for accounts payable

Any decrease in cash would be represented as negative amounts in this section.

It's important to note that the specific amounts and details would vary based on Peach Computer's individual financial transactions and operations. The operating activities section provides a summary of the cash inflows and outflows directly related to the company's core business operations during the specified period.

Learn more about revenue

brainly.com/question/8645356

#SPJ11

The profit (in dollars) from the sale of a palm trees is given by:
P(a) = 20x - 0.1x^2 - 100.
Find the profit at a sales of 13 trees

Answers

On a company's income statement, gross profit is computed by subtracting the cost of goods sold (COGS) from revenue. (sales),so the sale of palm tree is $143.10.

To find the profit from the sale of 13 palm trees, we need to substitute 13 for x in the profit function:

P(13) = 20(13) - 0.1(13)^2 - 100
P(13) = 260 - 16.9 - 100
P(13) = $143.10

Therefore, the profit from the sale of 13 palm trees is $143.10.

Visit here to learn more about sales

brainly.com/question/31406205

#SPJ11

The weight (in pounds) and height (in inches) for a child were measured every few months over a two-year period. The results are given in the table.



A 2-column table with 9 rows. Column 1 is labeled Weight (x) with entries 8, 12, 18, 24, 30, 32, 35, 37, 40. Column 2 is labeled Height (y) with entries 22, 23, 26, 30, 32, 33, 35, 36, 38.



Using technology, what is the correlation coefficient?



–0. 997


–0. 503


0. 503


0. 997

Answers

The correlation coefficient using technology is 0.997.

Using the given data in the table, the correlation coefficient can be calculated using technology, such as a statistical calculator or spreadsheet software.

Using python

import numpy as np

# Input the data

weight = np.array([8, 12, 18, 24, 30, 32, 35, 37, 40])

height = np.array([22, 23, 26, 30, 32, 33, 35, 36, 38])

# Calculate the correlation coefficient

correlation_coefficient = np.corrcoef(weight, height)[0, 1]

# Print the correlation coefficient

print("Correlation Coefficient:", correlation_coefficient)

The out put will be

Correlation Coefficient: 0.997088376189

The correlation coefficient (r) measures the strength and direction of a linear relationship between two variables, in this case, weight (x) and height (y) of a child.
Upon calculating, the correlation coefficient (r) is approximately 0.997. This indicates a strong positive linear relationship between the child's weight and height over the two-year period.

Corelation shows dependency of x on y variable and vice versa.

Learn more about correlation coefficient : https://brainly.com/question/12220189

#SPJ11

Evaluate the following expression. Your answer must be in exact form: for example, type pi/6 for π/6 or DNE if the expression is undefined. arcsin (sin (-57π/10))=

Answers

For the following expression arcsin (sin (-57π/10)) is 3π/10.

The function arcsin(x) gives the angle in radians whose sine is x.

In this problem, we need to find the angle whose sine is equal to the sine of -57π/10.

First, we need to simplify -57π/10 to an angle in the range [-π/2, π/2] since the sine function has a range of [-1, 1]. To do this, we use the fact that sine has a period of 2π,

which means that sin(-57π/10) = sin((-57π/10) + 4π) = sin(3π/10).

So we need to find the angle θ such that sin(θ) = sin(3π/10).

Since sine is an odd function, we know that sin(-θ) = -sin(θ), so we can also say that sin(θ) = sin(-3π/10).

Therefore, there are two possible angles that satisfy the equation: θ = 3π/10 or θ = -3π/10.

However, since the range of the arcsine function is [-π/2, π/2], only the angle in that range that satisfies the equation is θ = 3π/10.

Therefore, we can write:

arcsin(sin(-57π/10)) = arcsin(sin(3π/10)) = 3π/10

The answer is 3π/10.

To practice more questions on functions:

https://brainly.com/question/2328150

#SPJ11

The rate of change of the gender ratio for the United States during the twentieth century can be modeled as g(t) = (1. 68 · 10^−4)t^2 − 0. 02t − 0. 10


where output is measured in males/100 females per year and t is the number of years since 1900. In 1970, the gender ratio was 94. 8 males per 100 females.



(a) Write a specific antiderivative giving the gender ratio.


G(t) = _______________ males/100 females



(b) How is this specific antiderivative related to an accumulation function of g?


The specific antiderivative in part (a) is the formula for the accumulation function of g passing through (t, g) =

Answers

Answer:

(a) G(t) = (1.68 × 10^-4) × (1/3) t^3 - (0.02/2) t^2 - 0.10t - 2445.84

(b) A(t) = G(t) - G(1900) = (1.68 × 10^-4) × (1/3) (t^3 - 1900^3) - (0.02/2) (t^2 -     1900^2) - 0.10(t - 1900)

Step-by-step explanation:

(a) The antiderivative of g(t) can be found by integrating each term of the function with respect to t:

∫g(t) dt = ∫(1.68 × 10^-4)t^2 dt - ∫0.02t dt - ∫0.10 dt

= (1.68 × 10^-4) × (1/3) t^3 - (0.02/2) t^2 - 0.10t + C

where C is the constant of integration.

To find the specific antiderivative G(t) that passes through the point (1970, 94.8), we can use this point to solve for C:

94.8 = (1.68 × 10^-4) × (1/3) (1970)^3 - (0.02/2) (1970)^2 - 0.10(1970) + C

C = 94.8 + (1.68 × 10^-4) × (1/3) (1970)^3 - (0.02/2) (1970)^2 - 0.10(1970)

C ≈ -2445.84

Therefore, the specific antiderivative that gives the gender ratio is:

G(t) = (1.68 × 10^-4) × (1/3) t^3 - (0.02/2) t^2 - 0.10t - 2445.84

(b) The accumulation function of g is the integral of g with respect to t, or:

A(t) = ∫g(t) dt = G(t) + C

where C is the constant of integration. We can find the value of C using the initial condition given in the problem:

A(1900) = ∫g(t) dt ∣t=1900 = G(1900) + C = 0

Therefore, C = -G(1900), and the accumulation function of g is:

A(t) = G(t) - G(1900) = (1.68 × 10^-4) × (1/3) (t^3 - 1900^3) - (0.02/2) (t^2 - 1900^2) - 0.10(t - 1900)

To know more about antiderivative refer here

https://brainly.com/question/31385327#

#SPJ11

In a random sample of large cities around the world, the ozone level (in parts per million) and the population (in millions) were measured. Fitting the simple linear regression model gave the estimated regression equation: ozone⌢ = 8. 89 + 16. 6 population. (pretend it's a hat)



Interpret b = 16. 6. For each additional ________________________


million people, the predicted ozone level increases ___________________


ppm.



Rascoville is a large city with a population of 3 million people. What is the average ozone level? __________________________



If the ozone level is approximately 142 ppm, what is the approximate population in millions (round to the nearest million)? __________________________________

Answers

Interpretation:

The regression coefficient b = 16.6 represents the change in the predicted ozone level (in parts per million) for each additional million people in the population.

Specifically, for each additional million people, the predicted ozone level is expected to increase by 16.6 parts per million.

For Rascoville, a city with a population of 3 million people, we can use the estimated regression equation to predict the average ozone level:

ozone⌢ = 8.89 + 16.6 × 3 = 8.89 + 49.8 = 58.69

Therefore, the predicted average ozone level for Rascoville is 58.69 parts per million.

If the ozone level is approximately 142 ppm, we can use the estimated regression equation to estimate the population:

142 = 8.89 + 16.6 × population

Solving for population, we get:

133.11 = 16.6 × population

population ≈ 8.02 million

Therefore, the approximate population of the city is 8 million people (rounded to the nearest million).

To know more about ozone level refer here

https://brainly.com/question/29672449#

#SPJ11

ķojo and kofta were given 38000 to share. kojo had 7500 more than kofta find each of their shares Show working​

Answers

Answer:

Kofta receives $15,250 and Kojo receives $22,750.

Step-by-step explanation:

Let x represent the amount of money that Kofta has.

x + (x +7,500) = 38,000

          - 7,500   - 7,500

___________________

x + x = 30,500

2x = 30,500

÷ 2 = ÷2

-------------------

x = 15,250

Therefore, Kofta has $15,250.

Let k represent the amount of money that Kojo has.

k + 15,250 = 38,00

k = 38,000 - 15,250

k =  $22,750

Therefore, Kojo has $22,750

32


{(-0. 25, 2. 5), (1. 75, -5. 5), (3. 25, -11. 5)}


Write an equation in the form of y = mx + b that represents this linear function?

Answers

Therefore, the equation in the form of y = mx + b that represents this linear function is: y = -3.2x + 0.1

To write an equation in the form of y = mx + b for a linear function, we need to find the slope (m) and the y-intercept (b).

We can use any two points from the given set of points to find the slope:

m = (y2 - y1) / (x2 - x1)

Let's use the first and second points:

m = (-5.5 - 2.5) / (1.75 - (-0.25))

m = -8 / 2.5

m = -3.2

Now, we can use the slope and one of the points to find the y-intercept:

y = mx + b

-5.5 = (-3.2)(1.75) + b

b = -5.5 + 5.6

b = 0.1

To know more about linear function refer to

https://brainly.com/question/2248255

#SPJ11

A basketball coach wants to purchase shooting shirts for each member of a basketball team.


The cost of shooting shirts can be represented by the equation C = 0. 2x^2 + 1. 6x + 15, where


C is the amount it cost to purchase x shooting shirts. How many shooting shirts can the


basketball coach order for $300?


C = 2x? + 1. 6x + 15

Answers

The basketball coach can order approximately 34 shooting shirts for $300.

To determine the number of shooting shirts the basketball coach can order for $300, we need to solve the equation C = 0.2x^2 + 1.6x + 15, where C represents the cost and x represents the number of shooting shirts.

The equation is given as C = 0.2x^2 + 1.6x + 15.

To find the number of shooting shirts for $300, we set the cost C equal to 300 and solve for x:

0.2x^2 + 1.6x + 15 = 300

0.2x^2 + 1.6x + 15 - 300 = 0

0.2x^2 + 1.6x - 285 = 0

Now we can solve this quadratic equation using factoring, completing the square, or the quadratic formula. Let's use the quadratic formula:

x = (-b ± sqrt(b^2 - 4ac)) / (2a)

For this equation, a = 0.2, b = 1.6, and c = -285. Plugging in these values into the quadratic formula:

x = (-1.6 ± sqrt(1.6^2 - 4 * 0.2 * -285)) / (2 * 0.2)

Simplifying the equation further:

x = (-1.6 ± sqrt(2.56 + 228)) / 0.4

x = (-1.6 ± sqrt(230.56)) / 0.4

x = (-1.6 ± 15.18) / 0.4

Now we have two solutions:

x1 = (-1.6 + 15.18) / 0.4 = 33.95

x2 = (-1.6 - 15.18) / 0.4 = -44.95

Since the number of shooting shirts cannot be negative, we discard the negative solution.

Therefore, the basketball coach can order approximately 34 shooting shirts for $300.

Learn more about quadratic formula,

https://brainly.com/question/30164833

#SPJ11

Question 4 of 15


Cryshel is mailing pillows with a total volume of 9. 5 ft3. She needs a mailing


box that has a volume greater than 9. 5 ft.


• Box A: length = 3 ft, width = 2 ft, height = 1. 5 ft


• Box B: length = 2. 5 ft, width = 2 ft, height = 2 ft


Which box is large enough to hold all of her pillows?


O


A. Neither box


B. Both box A and box B


ОО


C. Box B


D. Box A

Answers

Answer:

  C.  Box B

Step-by-step explanation:

You want to know which of these two boxes has a volume greater than 9.5 ft³:

Box A: 3 ft by 2 ft by 1.5 ftBox B: 2.5 ft by 2 ft by 2 ft

Volume

The volume of each box is found by multiplying its dimensions:

  Box A: (3 ft)(2 ft)(1.5 ft) = 9 ft³

  Box B: (2.5 ft)(2 ft)(2 ft) = 10 ft³

Only box B is large enough, choice C.

<95141404393>

In what time will Rs. 6350 amounts to Rs. 8255 if the simple Interest is calculated at 10% per annum ?Also, find the rate of interst if He returns in 1 1/2 years .

Answers

Therefore, it will take 3 years for Rs. 6350 to amount to Rs. 8255 at 10% per annum. Therefore, the rate of interest is 32.6% per annum if the loan is returned in 1 1/2 years.

What is interest?

Interest is the amount of money that a lender charges a borrower for the use of money or assets. It is typically expressed as a percentage of the amount borrowed or invested, and is paid by the borrower to the lender as compensation for the use of the money. There are two main types of interest: simple interest and compound interest. Simple interest is calculated based on the initial amount borrowed or invested, and is paid out at regular intervals over a fixed period of time. Compound interest, on the other hand, is calculated based on the initial amount plus any accumulated interest, and is paid out at the end of the investment period.

Here,

Using the formula for simple interest:

Simple Interest = (Principal x Rate x Time) / 100

where Principal is the initial amount, Rate is the interest rate per annum, and Time is the duration of the loan in years. To find the time it takes for Rs. 6350 to amount to Rs. 8255 at 10% per annum, we can set up the equation:

8255 - 6350 = (6350 x 10 x Time) / 100

Simplifying the equation, we get:

1905 = 635 x Time

Time = 1905 / 635

Time = 3 years

To find the interest rate if the loan is returned in 1 1/2 years, we can rearrange the formula for simple interest as:

Rate = (100 x Simple Interest) / (Principal x Time)

Plugging in the values, we get:

Rate = (100 x (8255 - 6350)) / (6350 x 1.5)

Rate = 3105 / 9525

Rate = 0.326 or 32.6%

To know more about interest,

https://brainly.com/question/28792777

#SPJ1

Line x is parallel to line y. Line z intersect lines x and y. Determine whether each statement is Sometimes True.

Answers

Answer:

Step-by-step explanation:

a and b are sometimes true.

This is when line z intersects x and y at right angles.

HELP ASAP!!!!!!!!!!!

Answers

Answer:

25%

Step-by-step explanation:

The total number of 7th grade students = 9 + 11 + 11 + 13 = 44

Out of the 44 students 11 play bass

Probability that a seventh grader chosen at random will play the base is:

11/44 = 1/4 = 0.25

As a percentage, this would be 0.25 x 100 = 25%

(1 point) Evaluate the double integral I = s do xy dA where D is the triangular region with vertices (0,0),(1,0), (0,6).

Answers

To evaluate the double integral I = ∬D xy dA, where D is the triangular region with vertices (0,0),(1,0), (0,6), we need to set up the limits of integration for x and y.

Since D is a triangular region, we can integrate over the two sides that meet at the origin and then integrate over the third side. Let's integrate over the sides that form the right angle at (0,0).

For the side along the x-axis, y = 0 to y = 6x.

For the side along the y-axis, x = 0 to x = 1.

Thus, the double integral becomes:

I = ∫0^1 ∫0⁶x xy dy dx

Evaluating the inner integral with respect to y, we get:

I = ∫0^1 [x(y²/2)]0⁶x dx

Simplifying and evaluating the outer integral with respect to x, we get:

I = ∫0^1 18x⁴ dx

I = 18/5

Therefore, the value of the double integral I = ∬D xy dA over the triangular region with vertices (0,0),(1,0), (0,6) is 18/5.
To evaluate the double integral I = ∬_D xy dA for the triangular region D with vertices (0,0), (1,0), and (0,6), we first need to set up the limits of integration.

The base of the triangle lies on the x-axis, from x = 0 to x = 1. The height of the triangle lies on the y-axis, from y = 0 to the line y = 6(1-x), since the slope of the hypotenuse is -6 and passes through (1,0).

Now we can set up the integral:

I = ∬_D xy dA = ∫_(0 to 1) ∫_(0 to 6(1-x)) xy dy dx

Let's first integrate with respect to y:

∫_(0 to 6(1-x)) xy dy = [x(y²)/2]_(0 to 6(1-x)) = 18x(1-x)²

Next, integrate with respect to x:

I = ∫_(0 to 1) 18x(1-x)² dx

Using integration by substitution or expanding and integrating term by term, we get:

I = 2

So, the value of the double integral is 2.

Visit here to learn more about double integral brainly.com/question/27360126
#SPJ11

How do you solve the cube root function of x²/³ = 16?

Answers

The cube root of the given function is [tex]x=2\sqrt[3]{2}[/tex].

The given function is x³=16.

Here, the given function can be written as

[tex]x=\sqrt[3]{16}[/tex]

[tex]x=\sqrt[3]{2\times2\times2\times2}[/tex]

[tex]x=\sqrt[3]{2^3\times2}[/tex]

[tex]x=2\sqrt[3]{2}[/tex]

Therefore, the cube root of the given function is [tex]x=2\sqrt[3]{2}[/tex].

Learn more about the radical form here:

brainly.com/question/27272065.

#SPJ1

"Your question is incomplete, probably the complete question/missing part is:"

How do you solve the cube root function of x³=16.

A quantitative data set has mean 25 and standard deviation 2. At least what percentage of the observations lie between 19 and 31 ​?

Answers

At least 95% of the observations lie between 19 and 31.

To see why, we can use Chebyshev's theorem, which states that for any data set, regardless of the shape of the distribution, at least 1 - (1/k²) of the observations lie within k standard deviations of the mean. In this case,

we want to know the percentage of observations that lie within two standard deviations of the mean, since 19 and 31 are both two standard deviations away from the mean of 25.

So, we can use k = 2 in Chebyshev's theorem, which gives us:

1 - (1/2²) = 1 - (1/4) = 0.75

Therefore, at least 75% of the observations lie within two standard deviations of the mean. However, since we know the data set is normally distributed (since we know the mean and standard deviation), we can use the empirical rule,

which states that for normally distributed data, approximately 68% of the observations lie within one standard deviation of the mean, and approximately 95% of the observations lie within two standard deviations of the mean.

Therefore, we can conclude that at least 95% of the observations lie between 19 and 31.

To know more about Chebyshev's theorem, refer here:

https://brainly.com/question/30584845#

#SPJ11

The base and all three faces of a triangle pyramid are equilateral triangles with side lengths of 3ft. the height of each triangle is 2.6ft. what are the lateral area and the total surface area of the triangular pyramid?

Answers

The lateral area of the triangular pyramid is 11.7 sq ft and the total surface area is 15.6 sq ft.

To find the lateral area and total surface area of the triangular pyramid with base and faces as equilateral triangles, we can follow these steps:

1: Find the area of one equilateral triangle.

To find the area of an equilateral triangle with side length 3 ft and height 2.6 ft, we can use the formula:

Area = (1/2) × base × height

Area = (1/2) × 3 × 2.6 = 3.9 sq ft

2: Calculate the lateral area.

Since the pyramid has three equilateral triangles as faces, we can multiply the area of one triangle by 3 to find the lateral area:

Lateral Area = 3 × 3.9 = 11.7 sq ft

3: Calculate the total surface area.

The total surface area includes both the lateral area and the base area. Since the base is also an equilateral triangle with the same dimensions, we can simply add the area of the base to the lateral area to find the total surface area:

Total Surface Area = Lateral Area + Base Area = 11.7 + 3.9 = 15.6 sq ft

In conclusion, the lateral area is 11.7 sq ft and the total surface area is 15.6 sq ft.

Learn more about Surface area:

https://brainly.com/question/16519513

#SPJ11

can someone help me solve -2√180u²v

Answers

Answer:-12 with the weird checkmark symbol then a five inside the checkmark symbol times u squared v

Step-by-step explanation:

In ARST, r = 58 cm, m/S=48° and m/T=29°. Find the length of s, to the nearest
centimeter.

Answers

The value of length 's' is 44.3 cm

What is sine rule?

The sine rule states that if a, b and c are the lengths of the sides of a triangle, and A, B and C are the angles in the triangle; with A opposite a, etc., then a/sinA=b/sinB=c/sinC.

The measure of angle R = 180-( 48+29)

R = 180- 77

R = 103°

sinR/ r = sinS/s

sin103 / 58 = sin48/s

s × sin103 = 58 × sin48

s × 0.974 = 43.1

s = 43.1/0.974

s = 44.3 cm

therefore the value of length is is 44.3 cm

learn more about sine rule from

https://brainly.com/question/20839703

#SPJ1

Look at picture please

Answers

Based on the inequality, 24.5x > 162 + 4.25x, Trina must sell more than 8 units of the handmade vases to make a profit.

What is inequality?

Inequality refers to a mathematical statement that two or more algebraic expressions are unequal or inequivalent.

Mathematically, inequalities are depicted as:

Greater than (>)Greater than or equal to (≥)Less than (<)Less than or equal to (≤)Not equal to (≠).

Selling price per handmade vase = $24.50

Variable cost per unit = $4.25

Fixed selling cost = $162

Let the number of vases to sell to make a profit = x

The total sales revenue = 24.5x

The total cost = 162 + 4.25x

To make a profit, 24.5x must be greater than 162 + 4.25x.

Inequality:

24.5x > 162 + 4.25x

20.25x > 162

x > 8

Check:

Total sales revenue = $196 ($24.5(8)

Total cost = $196 ($162 + $4.25(8)

Learn more about inequalities at https://brainly.com/question/25944814.

#SPJ1

The diameter of Circle Q terminates on the circumference of the circle at (0,3)and (0,−4). Write the equation of the circle in standard form. Show all of your work.
Need answer ASAP Please!!

Answers

Answer:

[tex]x^2+\left(y+\dfrac{1}{2}\right)^2=\dfrac{49}{4}[/tex]

Step-by-step explanation:

The center of the circle is the midpoint of its diameter.

[tex]\boxed{\begin{minipage}{7.4 cm}\underline{Midpoint between two points}\\\\Midpoint $=\left(\dfrac{x_2+x_1}{2},\dfrac{y_2+y_1}{2}\right)$\\\\\\where $(x_1,y_1)$ and $(x_2,y_2)$ are the endpoints.\\\end{minipage}}[/tex]

Given the endpoints of the diameter are (0, 3) and (0, -4), to find the coordinates of the center of the circle, substitute the two endpoints into the midpoint formula:

[tex]\text{Center}=\left(\dfrac{0+0}{2},\dfrac{-4+3}{2}\right)=\left(0,-\dfrac{1}{2}\right)[/tex]

As the x-values of the endpoints of the diameter are the same, the length of the diameter, d, is the absolute value of the difference in y-values of the endpoints:

[tex]d=|3-(-4)|=7[/tex]

Therefore, the diameter of circle Q is 7 units.

The radius, r, of a circle is half its diameter. Therefore:

[tex]r=\dfrac{d}{2}=\dfrac{7}{2}[/tex]

[tex]\boxed{\begin{minipage}{4 cm}\underline{Equation of a circle}\\\\$(x-h)^2+(y-k)^2=r^2$\\\\where:\\ \phantom{ww}$\bullet$ $(h,k)$ is the center. \\ \phantom{ww}$\bullet$ $r$ is the radius.\\\end{minipage}}[/tex]

Now we have determined the center and radius of circle Q, we can substitute these values into the equation of a circle to write the equation of circle Q in standard form:

[tex](x-0)^2+\left(y-\left(-\dfrac{1}{2}\right)\right)^2=\left(\dfrac{7}{2}\right)^2[/tex]

[tex]x^2+\left(y+\dfrac{1}{2}\right)^2=\dfrac{49}{4}[/tex]

Therefore, the equation of circle Q in standard form is:

[tex]\boxed{x^2+\left(y+\dfrac{1}{2}\right)^2=\dfrac{49}{4}}[/tex]

The ratio of dolls thta jack had to peter was 5:2. After jack gave peter 15 dolls, they had the same amount of dolls. How many do they have together?

Answers

They have total of 70 dolls together.

Let the initial number of dolls Jack had be 5x and the number Peter had be 2x.


After Jack gave Peter 15 dolls, their amounts became equal. So, we can write the equation: 5x - 15 = 2x + 15

Now, solve the equation for x: 5x - 2x = 15 + 15, which simplifies to 3x = 30

Divide both sides by 3: x = 10

Now, find the initial number of dolls they had: Jack had 5x = 5(10) = 50 dolls, and Peter had 2x = 2(10) = 20 dolls.

After Jack gave Peter 15 dolls, both had 35 dolls (50 - 15 = 35, and 20 + 15 = 35).

So, together they have 35 + 35 = 70 dolls.

To learn more about number:  https://brainly.com/question/20400557

#SPJ11

Examine this system of equations. What integer should the first equation be multiplied by so that when the two equations are added together, the x term is eliminated?

StartFraction 1 Over 18 EndFraction + four-fifths y = 10

Negative five-sixths x minus three-fourths y = 3

Answers

Answer:

To solve this problem, we need to find an integer to multiply the first equation by so that when we add the two equations together, the x term is eliminated. Let's first rearrange the equations to make them easier to work with:

1/18 x + 4/5 y = 10

-5/6 x - 3/4 y = 3

To eliminate the x term, we need to multiply the first equation by a certain integer so that when we add it to the second equation, the x terms cancel out. To do this, we need to find a common multiple of the denominators of the x coefficients in both equations, which are 18 and -6. The least common multiple of 18 and -6 is 18, so we can multiply the first equation by 18:

18(1/18 x + 4/5 y = 10)

Simplifying this equation, we get:

x + 72/5 y = 180

Now we can add this equation to the second equation:

x + 72/5 y = 180

-5/6 x - 3/4 y = 3

Multiplying the second equation by 15 to get rid of the fractions, we get:

-25/2 x - 45/4 y = 45

Now we can add the two equations together to eliminate the x term:

-25/2 x + x + 72/5 y - 45/4 y = 180 + 45

Simplifying this equation, we get:

-13/20 y = 225/4

Multiplying both sides by -20/13, we get:

y = -450/13

Therefore, the integer we need to multiply the first equation by is 18, which corresponds to option B.

MARK AS BRAINLIEST!!!!!!!

A national grocery chain is considering expanding their selection of prepared meals available for purchase. They believe that nationwide, 67 percent of households purchase at least one prepared meal per week from the grocery store. The results of a survey given to a random sample of Maryland households found that 641 out of 1,035 households purchase at least one meal per week at the store

Answers

61.93% of the surveyed Maryland households purchase at least one prepared meal per week from the grocery store.

A national grocery chain is considering expanding their selection of prepared meals, and they believe that 67 percent of households purchase at least one meal per week from the grocery store. In a survey conducted in Maryland, 641 out of 1,035 households purchase at least one meal per week at the store.
To determine the percentage of Maryland households purchasing at least one prepared meal per week, follow these steps:
Divide the number of households purchasing at least one meal per week (641) by the total number of households surveyed (1,035).
Multiply the result by 100 to get the percentage.
Here's the calculation: (641 / 1,035) x 100 = 61.93%
So, 61.93% of the surveyed Maryland households purchase at least one prepared meal per week from the grocery store.

Learn more about surveyed here, https://brainly.com/question/196770

#SPJ11

Select the correct answer from each drop-down menu.
Coach Loren asks Kerry to analyze the batting percentages of players on the softball team.

Complete the sentences with the correct terms.

Coach Loren wants one number to describe how all of the values in the data set vary, so Kerry should tell her to use a measure of
.

Kerry could give her the
or the
of the data set.

Answers

Answer:

Kerry could give her the range or the standard deviation of the data set.

Step-by-step explanation:

Coach Loren wants one number to describe how all of the values in the data set vary, so Kerry should tell her to use a measure of variability.

Kerry could give her the range or the standard deviation of the data set.

Find the area of the surface extending upward from the circle x^2 + y^2 = 1 in the cy-plane to the plane z = 2 - x - y.

Answers

The area of the surface is π square units.

We can use a surface integral to find the area of the surface. The surface integral of a scalar function f over a surface S is given by:

∬S f dS

In this case, we want to find the area of the surface, so f = 1, and the integral reduces to:

∬S dS

We can parameterize the surface S using cylindrical coordinates:

x = r cosθ

y = r sinθ

z = 2 - r cosθ - r sinθ

The surface S is defined by the equation x^2 + y^2 = 1, which in cylindrical coordinates is r^2 = 1. Therefore, the surface integral becomes:

∬S dS = ∫∫R ||rθ|| dr dθ

where R is the region in the rθ-plane that corresponds to the surface S.

To find the limits of integration for r and θ, we need to determine the bounds of the region R. Since r^2 = 1, we have r = 1 for all θ. The region R is therefore a circle of radius 1 centered at the origin, and we can integrate over the full range of θ:

∫0^2π ∫0^1 r dr dθ = π

Therefore, the area of the surface is π square units.

To learn more about surface integral visit: https://brainly.com/question/15177673

#SPJ11

Find the measure of arc AD

Answers

90 + 63 = 153

this is because the pink square means that degree is 90°

6
The expression √532 + 46√3 is equivalent to the expression r + p, where r and p are positive
integers. What is the value of r + p?

Answers

The value of r+p in the expression is  2(√133 + 23√3)

To simplify the given expression, we need to first simplify the square root of 532.

We can factor 532 as 2 × 2 × 7 × 19, and then group the factors in pairs of two to simplify the square root:

√532 = √(2 × 2 × 7 × 19) = √(2 × 2) × √(7 × 19)

= 2√(7 × 19)

= 2√133

Now we can substitute this expression into the original expression and combine like terms:

√532 + 46√3

= 2√133 + 46√3

= 2√133 + 2(23√3)

= 2(√133 + 23√3)


To learn more on Expressions click:

https://brainly.com/question/14083225

#SPJ1

Other Questions
write 5-7 sentences minimum as to why this source supports the new deal. The ending for the book is a surprise. In your essay answer, describe the ending in detail. Include the pivotal or important characters, what happened, and if there is an indication of what will happen in Cadbury after the book ends. Sword of the rightful kingPlease help What does [OH-] equal when [H*] = 1. 15 x 108 M?721*10-5 M The table shows the number of devices owned by a local company. What percent of these devices are tablets? Laptop- 44Tablet- 94Desktop- 62 1. look at the first stanza of poe's poem. what simile does he use to describehelen's beauty? If your end product is 200. 0 g KMnO4 how much KOH did you start with? A quantity with an initial value of 8200 grows continuously at a rate of 0. 55% per decade. What is the value of the quantity after 97 years, to the nearest hundredth?Answer: 4000. 4 A school chess club needs to raise at least $750 to attend a state competition. When body temp increases it causesmetabolic reactions to increase, whichreleases heat that further increases bodytemp. This is an example ofa. Homeostasisb. Positive feedbackc. Negative feedbackd. All of the above Libby starts draining the pool for cleaning. The function y = 10,080 - 720x represents thegallons of water y remaining in the pool after x hours. Find the zero and explain what it means in the context of the situation How can a speech or public statement resonate differently with various audiences, depending upon their point of view? A sample of 60 Grade 9 students' ages was obtained to estimate the mean age of all Grade 9 students. Consider that X= 15. 3 years and the population variance is 16. (Note: Standard Deviation is the square root of variance). Assume that the distribution is normal. Answer the following questions:1. What is the point estimate for ?2. Find the 95% confidence interval for . 3. Find the 99% confidence interval for . 4. What conclusions can you make based on each interval estimate ? Question 1(Multiple Choice Worth 2 points)(Circle Graphs MC)The circle graph describes the distribution of preferred transportation methods from a sample of 400 randomly selected San Francisco residents.circle graph titled San Francisco Residents' Transportation with five sections labeled walk 40 percent, bicycle 8 percent, streetcar 15 percent, bus 10 percent, and cable car 27 percentWhich of the following conclusions can we draw from the circle graph?Together, Streetcar and Cable Car are the preferred transportation for 168 residents.Together, Walk and Streetcar are the preferred transportation for 55 residents.Bus is the preferred transportation for 45 residents.Bicycle is the preferred transportation for 50 residents.Question 2(Multiple Choice Worth 2 points)(Appropriate Measures MC)The box plot represents the number of tickets sold for a school dance.A horizontal line labeled Number of Tickets sold that starts at 8, with tick marks every one unit up to 30. The graph is titled Tickets Sold for A Dance. The box extends from 17 to 21 on the number line. A line in the box is at 19. The lines outside the box end at 10 and 27.Which of the following is the appropriate measure of center for the data, and what is its value?The mean is the best measure of center, and it equals 19.The median is the best measure of center, and it equals 4.The median is the best measure of center, and it equals 19.The mean is the best measure of center, and it equals 4.Question 3(Multiple Choice Worth 2 points)(Comparing Data LC)The histograms display the frequency of temperatures in two different locations in a 30-day period.A graph with the x-axis labeled Temperature in Degrees, with intervals 60 to 69, 70 to 79, 80 to 89, 90 to 99, 100 to 109, 110 to 119. The y-axis is labeled Frequency and begins at 0 with tick marks every one unit up to 14. A shaded bar stops at 10 above 60 to 69, at 9 above 70 to 79, at 5 above 80 to 89, at 4 above 90 to 99, and at 2 above 100 to 109. There is no shaded bar above 110 to 119. The graph is titled Temps in Sunny Town.A graph with the x-axis labeled Temperature in Degrees, with intervals 60 to 69, 70 to 79, 80 to 89, 90 to 99, 100 to 109, 110 to 119. The y-axis is labeled Frequency and begins at 0 with tick marks every one unit up to 16. A shaded bar stops at 2 above 60 to 69, at 4 above 70 to 79, at 12 above 80 to 89, at 6 above 90 to 99, at 4 above 100 to 109, and at 2 above 110 to 119. The graph is titled Temps in Desert Landing.When comparing the data, which measure of center should be used to determine which location typically has the cooler temperature?Median, because Desert Landing is symmetricMean, because Sunny Town is skewedMean, because Desert Landing is symmetricMedian, because Sunny Town is skewedQuestion 4(Multiple Choice Worth 2 points)(Appropriate Measures MC)A charity needs to report its typical donations received. The following is a list of the donations from one week. A histogram is provided to display the data.10, 11, 35, 39, 40, 42, 42, 45, 49, 49, 51, 51, 52, 53, 53, 54, 56, 59A graph titled Donations to Charity in Dollars. The x-axis is labeled 10 to 19, 20 to 29, 30 to 39, 40 to 49, and 50 to 59. The y-axis is labeled Frequency. There is a shaded bar up to 2 above 10 to 19, up to 2 above 30 to 39, up to 6 above 40 to 49, and up to 8 above 50 to 59. There is no shaded bar above 20 to 29.Which measure of variability should the charity use to accurately represent the data? Explain your answer.The range of 13 is the most accurate to use, since the data is skewed.The IQR of 49 is the most accurate to use to show that they need more money.The range of 49 is the most accurate to use to show that they have plenty of money.The IQR of 13 is the most accurate to use, since the data is skewed.Question 5(Multiple Choice Worth 2 points)(Making Predictions MC)A recent conference had 900 people in attendance. In one exhibit room of 80 people, there were 65 teachers and 15 principals. What prediction can you make about the number of principals in attendance at the conference?There were about 820 principals in attendance.There were about 731 principals in attendance.There were about 208 principals in attendance.There were about 169 principals in attendance.Question 6(Multiple Choice Worth 2 points)(Creating Graphical Representations LC)A teacher was interested in the subject that students preferred in a particular school. He gathered data from a random sample of 100 students in the school and wanted to create an appropriate graphical representation for the data.Which graphical representation would be best for his data?Stem-and-leaf plotHistogramCircle graphBox plot What are the two main factors that determine the choice of a network model How did greek writings affect feudalism? it strengthened the grip of feudal lords. it rejected the idea of kings and popes. it encouraged ideas about democracy in contrast to feudalistic rule. it proposed new ways to structure society and improve feudalism A standard piece of notebook paper measures 8.5 inches by 11 inches. By cutting a square out of each corner, the sides can be folded up to create a box with an open top. Determine the size of the square that needs to be cut out of each corner to create a box of maximum volume. For extra credit, perform this experiment from home and include a picture of the box you create. 3) (2 points) If f'(x)=6x 5 sin x+e and f(0) = 20, determine the function f. Find the exact solutions of the equation in the interval (0, 2). (Enter your answers as a comma-separated list) 4 tan 2x - 4 cot x = 0 x= /6 , /2, 5/6, 7/6, 3/2, 11/6 what are the answers to these questions? Where rmc enters a box, fitting, of other enclosure, _____ shall be provided to protect the wire from abrasion, unless the design of the box, fitting, or enclosure affords equivalent protection (1 point) Use implicit differentiation to find the points where the circle defined by x2 +y2-2x-4y = 11 has horizontal and vertical tangent lines. The circle has horizontal tangent lines at the point(s). The circle has vertical tangent lines at the point(s)